Forum: Digitale Signalverarbeitung / DSP / Machine Learning inverse z transformation


von keeke (Gast)


Lesenswert?

Hallo,

kann mir jemand bei folgender Aufgabe behilflich sein?

Gesucht ist die inverse z Transformation von

F(z)= z^2/(1/3z +1)

Ich habe eine Polynomdivision durchgeführt, da Nennergrad kleiner als 
der Zählergrad.

F(z) = -9/(1/3z+1) und einem Rest von z-3

Der Rest stört mich bei der Umwandlung... Kann mir jemand sagen, ob der 
Weg ünerhaupt stimmt?

von Paul (Gast)


Lesenswert?

also ich bekomme nach der polynomdivision 3(z-3) + 27/(z+3) raus

von keeke (Gast)


Lesenswert?

Ich habe so gerechnet:

z^2      : (z+3) = z-3
-(z^2+3z)
----------
-3z
-(-3z-9)
--------
9

von Paul (Gast)


Lesenswert?

ja meins ist exakt das selbe, außer dass du irgendwo den Faktor 3 
verschlürt hast ;)

von keeke (Gast)


Lesenswert?

Ok danke, aber ich habe immer noch keine Ahnung wie ich das 
transformiere.

von Paul (Gast)


Lesenswert?

Schau mal bei wikipedia. Das erste beispiel zur inversen z-trafo passt 
exakt zu deinem problem.

von keeke (Gast)


Lesenswert?

Du meinst das mit der Laurent-Reihe?
Ich kann das leider nicht nachvollziehen.
3(z-3) + 27/(z+3)

Also ich habe doch nicht folgende Form: z^n/(z-1)

Oder war die Polynomdivision umsonst?

von ich (Gast)


Lesenswert?

Muss man hier nicht einfach die ganze Kiste um 2 verschieben und dann 
passt es doch. (z hat für gewöhnlich negative Exponten). Der aktuelle 
Wert wird dann mit dem Wert der Vergangenheit berechnet.

von keeke (Gast)


Lesenswert?

Also rauskommen soll folgendes:
f(n)= (-3)^(n+2) u(-n-2)
Sieht wie du sagst um zwei verschoben aus. Allerdings wüsste ich gerne 
wie du darauf kommst.

von ich (Gast)


Lesenswert?

Vergesst es, es ist genau falsch herum.

von keeke (Gast)


Lesenswert?

Kann jemand denn die Lösung nachvollziehen?

von ich (Gast)


Lesenswert?

Ich würde es doch um 2 verschieben (Zähler = 1). Dann eine 
Partialbruchzerlegung durchführen. Dann solle man eigentlich eine 
Korrespondenz anwenden können, siehe Wiki.

von keeke (Gast)


Lesenswert?

Wie siehts du denn die Verschiebung um 2.

von keeke (Gast)


Lesenswert?

Achso weil der Zähler z^2 ist, wegen dem Exponenten?

von Michael W. (Gast)


Lesenswert?

Jetzt kenne ich mich nicht mehr aus...

Die z-Transformierte ist

Die Originalfunktion des ersten Terms ist

für

und die des zweiten, je nach Konvergenzbereich entweder

für

bzw.

für

http://de.wikipedia.org/wiki/Z-Transformation#Korrespondenzen

Korrespondenzen 2, 12, 13

keke sagt nichts über den Konvergenzbereich aus - wie soll da eine 
eindeutige Lösung rauskommen??

von keeke (Gast)


Lesenswert?

Konvergenzbereich ist |z|<3

von Michael W. (Gast)


Lesenswert?

hmm, dann geht meine Methode nicht...

von keeke (Gast)


Lesenswert?

Trotzdem danke und sorry, dass ich vergessen habe den Bereich anzugeben.

von Michael W. (Gast)


Lesenswert?

keeke schrieb:
> Trotzdem danke und sorry, dass ich vergessen habe den Bereich anzugeben.

nicht du musst dich entschuldigen...
jetzt bin ich als Beobachter nur gespannt, was rauskommt.
Ich arbeite immer mit Tabellen, die in 1/z angegeben sind.

Eine Korrespondenz für 1/(z+3) kann ich nirgends finden.

von Michael W. (Gast)


Lesenswert?

Sorry, dass ich das nochmals aufmache, aber ich verstehe immer noch 
nicht, wieso man bei dem von mir angegebenen Weg (der mir richtig 
erscheint, wenn auch nicht so elegant wie keeke's Lösung) ebenfalls auf


kommt, aber z=0 ausgeschlossen ist (siehe mein erstes Posting)


während man mit dem Verschiebungssatz, wie von keeke beschrieben auf das 
gleiche Ergebnis kommt, wobei z=0 aber eingeschlossen ist:

Die Konvergenzbereiche müssen sich ja überlappen...??? Immer wenn ich 
glaube etwas zu verstehen, kommt ein niederschmetterndes Ereignis, das 
mich an den Start zurückwirft und vermutlich eine schlaflose Nacht 
beschert ;-)

Aus der Sicht von keeke ist ja alles geklärt, jedoch ich habe mit der 
Diskrepanz immer noch ein Problem. Da ich jetzt keinen eigenen Thread 
aufmachen will, poste ich mein Anliegen hier als Ergänzung, und hoffe, 
eine Antwort zu bekommen.

von Michael W. (Gast)


Lesenswert?

Kann es sein, dass z=0 in meiner Aufspaltung des Originals eine 
hebbare Unstetigkeit ist, und es deshalb auch bei z=0 funktioniert?

von keeke (Gast)


Lesenswert?

Also verstanden habe ich das mit der Verschiebung nicht unbedingt.
Ich habe das Ganze als geometrische Reihe betrachtet und kam dann auf 
die Lösung :)

von Michael W. (Gast)


Lesenswert?

OK.

Für die Rücktransformation suchst du dir die passende Korrespondenz zu

und den angegebenen Konvergenzbereich |z|< 3,
multiplizierst mal 3 und wendest den Verschiebungssatz an, der sich aus 
der Multiplikation mit z ergibt. Du landest unmittelbar bei deiner 
angegebenen Lösung.

von keeke (Gast)


Lesenswert?

vielen dank.

Bitte melde dich an um einen Beitrag zu schreiben. Anmeldung ist kostenlos und dauert nur eine Minute.
Bestehender Account
Schon ein Account bei Google/GoogleMail? Keine Anmeldung erforderlich!
Mit Google-Account einloggen
Noch kein Account? Hier anmelden.